Help
I know why (E) is the answer, but I am still doubt why (C) couldn't be an answer for this questio...
Batman on December 29, 2014
  • December 2000 LSAT
  • SEC1
  • Q11
1
Reply
Help
Please explain how (C) weakens the argument that " the embezzler is one of the actuaries." Of cou...
Batman on December 26, 2014
  • October 2000 LSAT
  • SEC1
  • Q17
2
Replies
Help with question
Why isnt D the answer?
TheFacu on December 16, 2014
  • December 2010 LSAT
  • SEC3
  • Q10
1
Reply
Question
I'm not sure why the answer is what it is. If 26 percent would join and 16 percent would donate d...
Eunbeezy on December 12, 2014
  • December 2005 LSAT
  • SEC1
  • Q24
1
Reply
Help
Does "X is a subtype of disease Y" mean that X is dependent upon Y and can't affect without Y? I ...
Batman on December 11, 2014
  • June 1999 LSAT
  • SEC3
  • Q25
1
Reply
Please Explain
I do not understand the correct answer
KDA86 on December 10, 2014
  • June 2009 LSAT
  • SEC2
  • Q9
2
Replies
What's wrong with A?
I am having tough time to pick C over A. They both feels equally right...
Romans on December 6, 2014
  • December 2004 LSAT
  • SEC3
  • Q17
1
Reply
Help
Could you please explain why the answer goes to (d)? And, why (b) can't be the answer? Thanks,
Batman on December 6, 2014
  • June 1999 LSAT
  • SEC1
  • Q25
1
Reply
Need your explanation
Is the first sentence sufficient and necessary condition? If it is, Could you please write it out...
Batman on December 4, 2014
  • December 1998 LSAT
  • SEC1
  • Q20
1
Reply
Set-up and explanation help
Do you have any suggestions to answer this question in a quick process? Essentially the set-up of...
MJA7 on December 4, 2014
  • September 2007 LSAT
  • SEC2
  • Q23
1
Reply
Request your explanation
Although I got the right answer for this question,I didn't get a pretty much certain logical pict...
Batman on December 3, 2014
  • September 1998 LSAT
  • SEC3
  • Q22
1
Reply
Determining the conclusion help
I struggled to determine if the conclusion was about the unusual sparseness of Australia or the f...
MJA7 on December 2, 2014
  • June 2008 LSAT
  • SEC4
  • Q11
2
Replies
Incorrect answer choices
Answer choices D and E are not displayed correctly
jgrogreene on December 2, 2014
  • December 2013 LSAT
  • SEC2
  • Q20
1
Reply
Correct Answer?
Could you plz explain what lines this corresponds to?
ArashMash89 on December 2, 2014
  • June 2014 LSAT
  • SEC1
  • Q1
3
Replies
Help
I understand why the answer goes to (b). However, does commissioner really take for granted that ...
Batman on December 1, 2014
  • September 1998 LSAT
  • SEC2
  • Q17
1
Reply
Question
Why is the authors attitude towards the critics reasoned disagreement and not guarded agreement?
Eunbeezy on November 25, 2014
  • December 2004 LSAT
  • SEC3
  • Q13
1
Reply
Help
I have no idea why the answer is (d) and others are not answer. Isn't the answer of weakening qu...
Batman on November 25, 2014
  • June 1998 LSAT
  • SEC2
  • Q20
1
Reply
Please Explain
Please explain the correct answer further
KDA86 on November 25, 2014
  • September 2009 LSAT
  • SEC1
  • Q17
1
Reply
Explanations for diagnostic
Do you guys provide explanations for the diagnostic exams?
yoliarriaga on November 23, 2014
  • June 2009 LSAT
  • SEC4
  • Q1
2
Replies
Clarification
Why is "D" correct?
KDA86 on November 21, 2014
  • December 2004 LSAT
  • SEC5
  • Q14
1
Reply